GMAT Critical Reasoning Discussions

Puys help Me !

Q: Although the discount stores in Gorevilles central shopping district are expected to close within five years as a result of competition from a SpendLess discount department store that just opened, those locations will not stay vacant for long. In the five years since the opening of Colsons, a nondiscount department store, a new store has opened at the location of every store in the shopping district that closed because it could not compete with Colsons.

Which of the following, if true, most seriously weakens the argument?

A. Many customers of Colsons are expected to do less shopping there than they did before the SpendLess store opened.

B. Increasingly, the stores that have opened in the central shopping district since Colsons opened have been discount stores.

C. At present, the central shopping district has as many stores operating in it as it ever had.

D. Over the course of the next five years, it is expected that Gorevilles population will grow at a faster rate than it has for the past several decades.

E. Many stores in the central shopping district sell types of merchandise that are not available at either SpendLess or Colsons.

IMO : Option B

Data: Stores expected to close in 5yrs, because of SpendLess store.
Colson - a costly store is here since 5 yrs. Because of it, small discount stores come and go, regularly, as they cant compete.
Conclusion: Location of stores dont stay vacant.

To weaken, we need to find the reason that will EITHER increase the vacant stores OR reduce the frequency of opening and closing of stores in the district.

Now for the choices:
A - This will reduce Colson's customers, does not say where they will go and shop definitively.

B - If more and more discount stores are getting opened, more and more people will start preferring them over Colson's, as they have better prices. Once this happens, it will increase the customer base for these little stores, and they are less likely to close. So number of vacant spots will reduce more and more. When there are no vacant spots, we cant withhold the conclusion, as the prerequisite for the conclusion itself is now diminished.

C - Does not help in any way, as we know that the overall number of stores in the district has pretty much remained constant. One closes, another opens. People keep trying to compete with Colsons.

D - More population, more shopping. Does not mean it would help the little stores and their spots. May be more and more will be drawn towards SpendLess?

E - This one is a close contender, but product type is out of scope. And them selling products unavailable in C and S, does not justify them to linger around for ever.

---------
I went with a very detailed explanation on this one, as it is a weird arguement and definitely needs special attention. Usually CRs have 2 variables to deal with. This one has 3 - SpendLess, Colson, and little stores. And the conclusion is NOT about any of these, its about a 4th parameter called 'location spots'.

Very good one ponds_ggn !! OA Plz .....

IMO B.

Assumption of the author is that the trend since the opening of Colso's will follow the opening of Spendless store. New stores will still come up and spots will not remain vacant for long.
We need to break the assumption. B says that more and more discount stores came up around Colson's which does makes sense because Colson's isnt a discount store. Those who were non-discount stores shut down bec of the competition from Colson's
Now we have Spendless coming up which from the frst sentence of the argument indicates to give competition to discount stores. So in that vicinity, there is a very competent discount store as well as a non discount store which makes the situation quite unlikely to attract any new store.

p.s. took me around 5 mins to fig out. I hope I am not wrong atleast :D


Puys help Me !

Q: Although the discount stores in Goreville's central shopping district are expected to close within five years as a result of competition from a SpendLess discount department store that just opened, those locations will not stay vacant for long. In the five years since the opening of Colson's, a nondiscount department store, a new store has opened at the location of every store in the shopping district that closed because it could not compete with Colson's.

Which of the following, if true, most seriously weakens the argument?

A. Many customers of Colson's are expected to do less shopping there than they did before the SpendLess store opened.

B. Increasingly, the stores that have opened in the central shopping district since Colson's opened have been discount stores.

C. At present, the central shopping district has as many stores operating in it as it ever had.

D. Over the course of the next five years, it is expected that Goreville's population will grow at a faster rate than it has for the past several decades.

E. Many stores in the central shopping district sell types of merchandise that are not available at either SpendLess or Colson's.



I encountered this question on the GMAT PREP1 test and got it wrong -:(

In the year following an eight-cent increase in the federal tax on a pack of cigarettes, sales of cigarettes fell ten percent. In contrast, in the year prior to the tax increase, sales had fallen one percent. The volume of cigarette sales is therefore strongly related to the after-tax price of a pack of cigarettes.

Which of the following, if true, could most strengthen the argument above?

(A) During the second year after the tax increase, cigarette sales increased by a significant amount.
(B) The information available to consumers on the health risks of smoking remained largely unchanged in the period before and after the tax increase.
(C) Most consumers were unaware that the tax on cigarettes was going to increase.
(D) During the year following the cigarette tax increase, many consumers had less income, in inflation-adjusted dollars, than they had had in the previous year.
(E) During the year after the tax increase, there was a greater variety of cigarettes on the markey than there had been during the previous year.



I encountered this question on the GMAT PREP1 test and got it wrong -:(

In the year following an eight-cent increase in the federal tax on a pack of cigarettes, sales of cigarettes fell ten percent. In contrast, in the year prior to the tax increase, sales had fallen one percent. The volume of cigarette sales is therefore strongly related to the after-tax price of a pack of cigarettes.

Which of the following, if true, could most strengthen the argument above?

(A) During the second year after the tax increase, cigarette sales increased by a significant amount.
(B) The information available to consumers on the health risks of smoking remained largely unchanged in the period before and after the tax increase.
(C) Most consumers were unaware that the tax on cigarettes was going to increase.
(D) During the year following the cigarette tax increase, many consumers had less income, in inflation-adjusted dollars, than they had had in the previous year.
(E) During the year after the tax increase, there was a greater variety of cigarettes on the markey than there had been during the previous year.

IMO: Option B

Logic: Cause - tax increase, Effect - Decrease in Ciggy sales.
Methodology: To Weaken the argument - find other reasons that are compelling. To strengthen the argument - find other reasons that have minimal effect.

Now attack the choices:
A - Second year's influence does not conform nor deny the tax effect.
B - Other factors that can cause awareness and health importance, remained a constant. So correlation of Tax to price and analogously to sales in reaffirmed.
C - Consumers would be unaware only if there was no change in price. Negates the premise. so wrong.
D - Less income does not mean less ciggy sales. We all know smoking is addictive and people will find ways and means to hold on to their necessity.
E - More variety should only bolster sales, not reduce it. So wrong.

I would have marked option B

mostly the most subtle and non-extreme options are correct.. ;)




I encountered this question on the GMAT PREP1 test and got it wrong -:(

In the year following an eight-cent increase in the federal tax on a pack of cigarettes, sales of cigarettes fell ten percent. In contrast, in the year prior to the tax increase, sales had fallen one percent. The volume of cigarette sales is therefore strongly related to the after-tax price of a pack of cigarettes.

Which of the following, if true, could most strengthen the argument above?

(A) During the second year after the tax increase, cigarette sales increased by a significant amount.
(B) The information available to consumers on the health risks of smoking remained largely unchanged in the period before and after the tax increase.
(C) Most consumers were unaware that the tax on cigarettes was going to increase.
(D) During the year following the cigarette tax increase, many consumers had less income, in inflation-adjusted dollars, than they had had in the previous year.
(E) During the year after the tax increase, there was a greater variety of cigarettes on the markey than there had been during the previous year.

I will go with option B for the above CR




I encountered this question on the GMAT PREP1 test and got it wrong -:(

In the year following an eight-cent increase in the federal tax on a pack of cigarettes, sales of cigarettes fell ten percent. In contrast, in the year prior to the tax increase, sales had fallen one percent. The volume of cigarette sales is therefore strongly related to the after-tax price of a pack of cigarettes.

Which of the following, if true, could most strengthen the argument above?

(A) During the second year after the tax increase, cigarette sales increased by a significant amount.-weaken
(B) The information available to consumers on the health risks of smoking remained largely unchanged in the period before and after the tax increase.--not relevant to the price of cigg and tax
(C) Most consumers were unaware that the tax on cigarettes was going to increase.--not relevant
(D) During the year following the cigarette tax increase, many consumers had less income, in inflation-adjusted dollars, than they had had in the previous year.--strengthen.The statement attributes to the increase in percentage of drop in sales of cigg.
(E) During the year after the tax increase, there was a greater variety of cigarettes on the market than there had been during the previous year.-weaken.The greater variety of cigg. in the year after tax might have provided consumers more options.

I have marked my answer in bold.

IMO B
reasoning in red.




I encountered this question on the GMAT PREP1 test and got it wrong -:(

In the year following an eight-cent increase in the federal tax on a pack of cigarettes, sales of cigarettes fell ten percent. In contrast, in the year prior to the tax increase, sales had fallen one percent. The volume of cigarette sales is therefore strongly related to the after-tax price of a pack of cigarettes.

Which of the following, if true, could most strengthen the argument above?

(A) During the second year after the tax increase, cigarette sales increased by a significant amount. irrelevant
(B) The information available to consumers on the health risks of smoking remained largely unchanged in the period before and after the tax increase. This information, though not totally , comprehends that there was no other factor diminishing the sale.
(C) Most consumers were unaware that the tax on cigarettes was going to increase. it ensures there was a normal buying tendency. doesnt add anything to strengthen the argument
(D) During the year following the cigarette tax increase, many consumers had less income, in inflation-adjusted dollars, than they had had in the previous year. This weakens the argument. If people had less income , chances are they will cut down on cig. One notable point here is that, the argument mentions only an increase in tax on cig.
(E) During the year after the tax increase, there was a greater variety of cigarettes on the markey than there had been during the previous year. irrelevant



I encountered this question on the GMAT PREP1 test and got it wrong -:(

In the year following an eight-cent increase in the federal tax on a pack of cigarettes, sales of cigarettes fell ten percent. In contrast, in the year prior to the tax increase, sales had fallen one percent. The volume of cigarette sales is therefore strongly related to the after-tax price of a pack of cigarettes.

Which of the following, if true, could most strengthen the argument above?

(A) During the second year after the tax increase, cigarette sales increased by a significant amount.
(B) The information available to consumers on the health risks of smoking remained largely unchanged in the period before and after the tax increase.
(C) Most consumers were unaware that the tax on cigarettes was going to increase.
(D) During the year following the cigarette tax increase, many consumers had less income, in inflation-adjusted dollars, than they had had in the previous year.
(E) During the year after the tax increase, there was a greater variety of cigarettes on the markey than there had been during the previous year.



IMO - D
None of the other options strengthen the point of the author, i.e the after tax price affects sales!!
Both the options are scratchy.However,A looks better than E due to the following reason :

A says that the cost of manufacturing cloth has increased.
This will lead to a increase in the price of the cloth which we would buy ( though may depend on other factors as well ).One has to put Business Psyche here that why would I reduce the price of the finished good( just because I have raw material at a cheaper rate? )

E says that says the prices of harvesting cotton has gone increased.
One could question that since the prices of cotton have gone down..we don't know at what price they have sold the raw material to the manufacturers.Chances are likely tht they are sold at the same market price.

PS:They in E = farmers.

Any takers?

Regards,
Papa Ji

PG Rocks!


I would boil this down to A and E too ... but I choose A because E falls beyond the confines of the argument, while A falls within the confines of the argument.

the argument starts with the raw price of cotton and goes on to talk abt retail price of cotton.

processing of cotton takes place within these 2 events.

And harvesting takes place before the raw cotton price can be discussed, therefore, IMO, it lies beyond the confines of the argument.

hence A.



I encountered this question on the GMAT PREP1 test and got it wrong -:(

In the year following an eight-cent increase in the federal tax on a pack of cigarettes, sales of cigarettes fell ten percent. In contrast, in the year prior to the tax increase, sales had fallen one percent. The volume of cigarette sales is therefore strongly related to the after-tax price of a pack of cigarettes.

Which of the following, if true, could most strengthen the argument above?

(A) During the second year after the tax increase, cigarette sales increased by a significant amount.
(B) The information available to consumers on the health risks of smoking remained largely unchanged in the period before and after the tax increase.
(C) Most consumers were unaware that the tax on cigarettes was going to increase.
(D) During the year following the cigarette tax increase, many consumers had less income, in inflation-adjusted dollars, than they had had in the previous year.
(E) During the year after the tax increase, there was a greater variety of cigarettes on the markey than there had been during the previous year.


IMO : D

someone above mentioned my exact same reasoning, I just want to know the OA now πŸ˜ƒ

Ponds, whats the OA buddy?

Puys help Me !

Q: Although the discount stores in Gorevilles central shopping district are expected to close within five years as a result of competition from a SpendLess discount department store that just opened, those locations will not stay vacant for long. In the five years since the opening of Colsons, a nondiscount department store, a new store has opened at the location of every store in the shopping district that closed because it could not compete with Colsons.

Which of the following, if true, most seriously weakens the argument?

A. Many customers of Colsons are expected to do less shopping there than they did before the SpendLess store opened.

B. Increasingly, the stores that have opened in the central shopping district since Colsons opened have been discount stores.

C. At present, the central shopping district has as many stores operating in it as it ever had.

D. Over the course of the next five years, it is expected that Gorevilles population will grow at a faster rate than it has for the past several decades.

E. Many stores in the central shopping district sell types of merchandise that are not available at either SpendLess or Colsons.
IMO B.

Assumption of the author is that the trend since the opening of Colso's will follow the opening of Spendless store. New stores will still come up and spots will not remain vacant for long.
We need to break the assumption. B says that more and more discount stores came up around Colson's which does makes sense because Colson's isnt a discount store. Those who were non-discount stores shut down bec of the competition from Colson's
Now we have Spendless coming up which from the frst sentence of the argument indicates to give competition to discount stores. So in that vicinity, there is a very competent discount store as well as a non discount store which makes the situation quite unlikely to attract any new store.

p.s. took me around 5 mins to fig out. I hope I am not wrong atleast :D


Puys,
OA is Option-B
IMO B.

Assumption of the author is that the trend since the opening of Colso's will follow the opening of Spendless store. New stores will still come up and spots will not remain vacant for long.
We need to break the assumption. B says that more and more discount stores came up around Colson's which does makes sense because Colson's isnt a discount store. Those who were non-discount stores shut down bec of the competition from Colson's
Now we have Spendless coming up which from the frst sentence of the argument indicates to give competition to discount stores. So in that vicinity, there is a very competent discount store as well as a non discount store which makes the situation quite unlikely to attract any new store.

p.s. took me around 5 mins to fig out. I hope I am not wrong atleast :D

Puys help Me !

Q: Although the discount stores in Gorevilles central shopping district are expected to close within five years as a result of competition from a SpendLess discount department store that just opened, those locations will not stay vacant for long. In the five years since the opening of Colsons, a nondiscount department store, a new store has opened at the location of every store in the shopping district that closed because it could not compete with Colsons.

Which of the following, if true, most seriously weakens the argument?

A. Many customers of Colsons are expected to do less shopping there than they did before the SpendLess store opened.

B. Increasingly, the stores that have opened in the central shopping district since Colsons opened have been discount stores.

C. At present, the central shopping district has as many stores operating in it as it ever had.

D. Over the course of the next five years, it is expected that Gorevilles population will grow at a faster rate than it has for the past several decades.

E. Many stores in the central shopping district sell types of merchandise that are not available at either SpendLess or Colsons.


Hi,
OA is Option-B.
Hi,
OA is Option-B.


That was a nice explanation.
We got to target some fixed number of problems each day...i feel..

Hello,

For the CR strehgthening and weakening an arguments questions, is it that for weakening one needs to attack an assumption of an argument by negating it and for strengthening, one needs to consolidate the evidence or conclusion ?

Hello,

For the CR strehgthening and weakening an arguments questions, is it that for weakening one needs to attack an assumption of an argument by negating it and for strengthening, one needs to consolidate the evidence or conclusion ?

This asks you to identity the answer choice that best supports the argument, The Correct answer does not necessarily justify the argument, nor is the correct answer necessarily an assumption. The correct answer helps the answer choice in some way.

You are strengthening the conclusion.
Ask yourself if the answer choice will assist in some way.

This question type requires you to find the missing link between a premise and the conclusion (similar to finding the assumption), which when found will strengthen the conclusion.
Rules:
1. Stimulus has an Argument
2. Focus on the conclusion. All answers impact the conclusion.

Strengthen:
Questions ask you to support the argument in anyway possible
Uses words such as: Support, strengthen, helps, justifies.

How to strengthen:
- Find an answer that helps the authors conclusion in some way.
For Causality and Strengthening questions:
The answer will
- Eliminate any alternate causes for the stated effect. One cause should be
- Show that when the cause occurs the effect occurs and vice versa
- No reversed relationship

Weakening Question
When you are weakening you are targeting the assumption (Undermine). These types of question ask you to undermine the argument stated by the conclusion.
Here is were the answer choice affect the Stimuli
For these questions an argument will always be present with a conclusion attached. Therefore, know the specifics of the conclusion.
Identification words for Weakening: Refute, attack, undermine, argue against, cast doubt, challenge, damage, counter, call into question, weaken.

How to weaken an Argument:
  1. Always ask (when choosing the correct answer), if this choice will make the author reconsider his position or respond to something.
  2. Look at the conclusion, and while keeping the assumption in mind, find something that takes away from what is said.
  3. Weakening Scenarios:
    1. The author fails to consider all the possibilities
    2. Comparison of items that are different (flaw)
    3. Open conclusion


Conclusion: Look to see whether the Premise support the conclusion properly.
      1. Find the assumption- The weakener will challenge the assumption. USE THE DEFENDER RULE USE OF NOT
      2. The correct answer will show that that conclusion fails to account for something or does not follow from the stated premise. The author may fail to consider other possibilities, if you realize this and an answer choice points it out, that is the correct choice. Also, there may be an improper comparison made. The argument may even be left open for attack.


The wrong answer choice would be to choose an answer that is completely opposite of what is stated. Also Out of scope answers.

Need help with this one, kindly provide an explanation as well!




Free public education is the best form of education there is. Therefore, we must fight to ensure its continued existence; that is, we must be ready to defend the principle of equality of educational opportunity. Because this principle is we worth defending, it is clear that free public education is better than any other form of education. Which one of the following illustrates the same weak reasoning as found in the passage?

  1. (A) I love music, and thats why I listen to it constantly. I have my stereo or radio on every waking minute. Since I play music all the time, I must really love it.

  1. (B) Books are my most valuable possessions. My books are like my friendseach pleases me in different ways. Just as I would give up everything to save my friends, so too with my books.

  1. (C) I would much rather be poor and respected than be rich and despised. To have the respect of others is far more valuable than to have millions of dollars.

  1. (D) I have never been betrayed by any of my friends. They have been true to me through good times and bad. Therefore I will never betray any of my friends.

  1. (E) Because every plant I have ever seen has green leaves, I have concluded that all plants must have green leaves. This looks like a plant but it does not have green leaves, so it cannot be a plant.



And another one!


Which of the following, if true, is the most logical completion of the argument below? The tax system of the Republic of Grootland encourages borrowing by granting its taxpayers tax relief for interest paid on loans. The system also discourages saving by taxing any interest earned on savings. Nevertheless, it is clear that Grootlands tax system does not consistently favor borrowing over saving, for if it did, there would be no______

  1. (A) tax relief in Grootland for those portions of a taxpayers income, if any, that are set aside to increase that taxpayers total savings

  1. (B) tax relief in Grootland for the processing fees that taxpayers pay to lending institutions when obtaining certain kinds of loans

  1. (C) tax relief in Grootland for interest that taxpayers are charged on the unpaid balance in credit card accounts

  1. (D) taxes due in Grootland on the cash value of gifts received by taxpayers from banks trying to encourage people to open savings accounts

  1. (E) taxes due in Grootland on the amount that a taxpayer has invested in interest-bearing savings accounts


Cheers!
Need help with this one, kindly provide an explanation as well!




Free public education is the best form of education there is. Therefore, we must fight to ensure its continued existence; that is, we must be ready to defend the principle of equality of educational opportunity. Because this principle is we worth defending, it is clear that free public education is better than any other form of education.
Which one of the following illustrates the same weak reasoning as found in the passage?

  1. (A) I love music, and that's why I listen to it constantly. I have my stereo or radio on every waking minute. Since I play music all the time, I must really love it. - "Not a Contender"....X

  1. (B) Books are my most valuable possessions. My books are like my friends-each pleases me in different ways. Just as I would give up everything to save my friends, so too with my books.

  1. (C) I would much rather be poor and respected than be rich and despised. To have the respect of others is far more valuable than to have millions of dollars. - "Extreme"....X

  1. (D) I have never been betrayed by any of my friends. They have been true to me through good times and bad. Therefore I will never betray any of my friends. - X never cause Y so Y shall never cause X...No parallelism with reasoning

  1. (E) Because every plant I have ever seen has green leaves, I have concluded that all plants must have green leaves. This looks like a plant but it does not have green leaves, so it cannot be a plant.
This is my call..

Question of parallel reasoning:
there are a lot of Arguments and conclusions, offcourse there are assumptions madw which are wrong and a simlar thing is required to do here
Premise - 1:
-------
>> Free public education is the best form of education there
Intermediate conclusion:
------------------------
Therefore, we must fight to ensure its continued existence; that is, we must be ready to defend the principle of equality of educational opportunity
>> Please note that it is meant to serve here as an additional premise for the final argument and thats the catch
Premise - 2:
---------
>> Because this principle is we worth defending
Conclusion
----------
>> free public education is better than any other form of education
And another one!

Paraphrasing reveals:
"Because free education is best and provides oppertunity of equality in education, we must defend it. then it ends up saying," its better than any other form of the education" ...there is a huge assumption and that has to be picked up

Which of the following, if true, is the most logical completion of the argument below?
The tax system of the Republic of Grootland encourages borrowing by granting its taxpayers tax relief for interest paid on loans. The system also discourages saving by taxing any interest earned on savings. Nevertheless, it is clear that Grootland's tax system does not consistently favor borrowing over saving, for if it did, there would be no______

  1. (A) tax relief in Grootland for those portions of a taxpayer's income, if any, that are set aside to increase that taxpayer's total savings

  1. (B) tax relief in Grootland for the processing fees that taxpayers pay to lending institutions when obtaining certain kinds of loans

  1. (C) tax relief in Grootland for interest that taxpayers are charged on the unpaid balance in credit card accounts

  1. (D) taxes due in Grootland on the cash value of gifts received by taxpayers from banks trying to encourage people to open savings accounts (The catch is word: Consistently in premise)

  1. (E) taxes due in Grootland on the amount that a taxpayer has invested in interest-bearing savings accounts
Cheers!


I'm little confuse in D/E, but will pick D...
well, what the heck...this is really tough...
Need help with this one, kindly provide an explanation as well!




Free public education is the best form of education there is. Therefore, we must fight to ensure its continued existence; that is, we must be ready to defend the principle of equality of educational opportunity. Because this principle is we worth defending, it is clear that free public education is better than any other form of education. Which one of the following illustrates the same weak reasoning as found in the passage?

  1. (A) I love music, and that's why I listen to it constantly. I have my stereo or radio on every waking minute. Since I play music all the time, I must really love it.

  1. (B) Books are my most valuable possessions. My books are like my friends-each pleases me in different ways. Just as I would give up everything to save my friends, so too with my books.

  1. (C) I would much rather be poor and respected than be rich and despised. To have the respect of others is far more valuable than to have millions of dollars.

  1. (D) I have never been betrayed by any of my friends. They have been true to me through good times and bad. Therefore I will never betray any of my friends.

  1. (E) Because every plant I have ever seen has green leaves, I have concluded that all plants must have green leaves. This looks like a plant but it does not have green leaves, so it cannot be a plant.



And another one!


Which of the following, if true, is the most logical completion of the argument below? The tax system of the Republic of Grootland encourages borrowing by granting its taxpayers tax relief for interest paid on loans. The system also discourages saving by taxing any interest earned on savings. Nevertheless, it is clear that Grootland's tax system does not consistently favor borrowing over saving, for if it did, there would be no______

  1. (A) tax relief in Grootland for those portions of a taxpayer's income, if any, that are set aside to increase that taxpayer's total savings

  1. (B) tax relief in Grootland for the processing fees that taxpayers pay to lending institutions when obtaining certain kinds of loans

  1. (C) tax relief in Grootland for interest that taxpayers are charged on the unpaid balance in credit card accounts

  1. (D) taxes due in Grootland on the cash value of gifts received by taxpayers from banks trying to encourage people to open savings accounts

  1. (E) taxes due in Grootland on the amount that a taxpayer has invested in interest-bearing savings accounts


Cheers!


Hi speedwagon,

My choices are in color, gawd I'm still not sure about these..:banghead:
1) Defending interest in music similar to free education, both must be protection by fighting or indulgence.

2)clue is" borrowing over savings" favor.

What's OA?